subject
Mathematics, 08.07.2020 01:01 abemorales

If m ≤ f(x) ≤ M for a ≤ x ≤ b, where m is the absolute minimum and M is the absolute maximum of f on the interval [a, b], then m(b − a) ≤ b f(x) dx a ≤ M(b − a). Use this property to estimate the value of the integral. π/9 5 tan(3x) dx π/12 (smaller value) (larger value)

ansver
Answers: 2

Another question on Mathematics

question
Mathematics, 21.06.2019 16:20
7.(03.01 lc)which set represents the range of the function shown? {(-1, 5), (2,8), (5, 3), 13, -4)} (5 points){-1, 2, 5, 13){(5, -1), (8, 2), (3,5), (-4, 13)){-4, 3, 5, 8}{-4, -1, 2, 3, 5, 5, 8, 13}
Answers: 3
question
Mathematics, 21.06.2019 21:00
How do i put 3(x+7) in distributive property
Answers: 2
question
Mathematics, 21.06.2019 22:00
There are 3232 forwards and 8080 guards in leo's basketball league. leo must include all players on a team and wants each team to have the same number of forwards and the same number of guards. if leo creates the greatest number of teams possible, how many guards will be on each team?
Answers: 2
question
Mathematics, 21.06.2019 22:30
What is the least common multiple for 6 and 8? what is the least common multiple for 4 and 12 ? what is the least common multiple for 11 and 12? what is the least common multiple for 3 and 6?
Answers: 1
You know the right answer?
If m ≤ f(x) ≤ M for a ≤ x ≤ b, where m is the absolute minimum and M is the absolute maximum of f on...
Questions
Questions on the website: 13722363